Bound on difference between irrational square root and rational number

Solution 1:

I suppose that $d\in \mathbb{N}$, $d\geq 2$. Let $p/q$ a positive rational, $p,q\in \mathbb{N}$.

Suppose first that $\displaystyle |\sqrt{d}-p/q|\leq 1$. Then we get $\displaystyle p/q\leq \sqrt{d}+1$, and we have $\displaystyle \sqrt{d}+p/q\leq 2\sqrt{d}+1$.

Now:

$$|\sqrt{d}-p/q|=\frac{|q^2d-p^2|}{q^2(\sqrt{d}+p/q)}\geq \frac{|q^2d-p^2|}{q^2(2\sqrt{d}+1)} $$

Now $q^2 d-p^2 $ is not $0$, in $\mathbb{Z}$, hence $\displaystyle |q^2 d-p^2|\geq 1$, and $\displaystyle |\sqrt{d}-p/q|\geq \frac{1}{q^2(2\sqrt{d}+1)} $

If $\displaystyle |\sqrt{d}-p/q|\geq 1$, then obviously $\displaystyle |\sqrt{d}-p/q|\geq \frac{1}{q^2(2\sqrt{d}+1)}$, so this inequality is true for all $p,q$.

Solution 2:

$\sqrt2$ Your problem is part of a glorious history in mathematics. This begins with Liouville, who showed in 1844 that for every irrational algebraic $\alpha$ of degree $\ge 2$, there is a constant $c=c(\alpha)\gt 0$ such that $|\alpha-\frac {p}{q}|>$ $\frac{c}{q^n}$ for all rational $\frac{p}{q}\space (q\gt 0)$ (your post is the particular case $n=2$ and you can find the proof for n in many books of diophantine approximation).

His theorem allows Liouville to find the first known transcendental number and states, in other words, that the measure of irrationality of the irrational algebraic $\alpha$ cannot be greater than its degree.

This measure of irrationality of $\alpha$ it's pretty smaller but it had to wait until 1955 to be known, grace to Klaus Friedrich Roth whose work it is a pinnacle of mathematics.

The exponent $n$ of Liouville was improved on several important occasions which we summarize below.

Joseph Liouville (1809-1882): exponent n (1844).

Axel Thue (1863-1922): exponent $\frac {n}{2} +1$ (1909).

Carl Ludwig Siegel (1896-1981): exponent $2\sqrt n$ (1929).

Freeman John Dyson (1923- ): exponent $\sqrt{2n}$ (1947).

Klaus Friedrich Roth (1925- ): exponent 2 (1955). The best exponent. A deep result which earned its author the Fields Medal.

"The achievement is one that speaks for itself: it closes a chapter, and a new chapter is now opened. Roth’s theorem settles a question which is both of a fundamental nature and of extreme difficulty. It will stand as a landmark in mathematics for as long as mathematics is cultivated" (Harold Davenport’speech on the presentation to the Fields Medal at the International Congress of Edinburgh, 1958).

enter image description here

Solution 3:

Let $ d $ be a positive integer which isn't a square, and consider a rational $ \frac{p}{q} $ ($ p, q $ integers, $ q > 0 $).

Looking at $ \left| \sqrt{d} - \frac{p}{q} \right| $, it is $ \dfrac{1}{q} |q\sqrt{d} - p | $ $ = \dfrac{1}{q} \dfrac{|q^2 d - p^2|}{|q\sqrt{d}+p|} $. As $ \sqrt{d} $ is irrational, numerator $ |q^2 d - p^2 | $ is non-zero (and an integer), making $ | q^2 d - p^2 | \geq 1 $.

So $ \left| \sqrt{d} - \dfrac{p}{q} \right| \geq \dfrac{1}{q^2} \dfrac{1}{\left| \sqrt{d} + \frac{p}{q} \right|}$. Notice denominator $$\left| \sqrt{d} + \frac{p}{q} \right|= \left| 2\sqrt{d} -\sqrt{d} + \frac{p}{q} \right|\leq 2\sqrt{d} + \left| -\sqrt{d} + \frac{p}{q} \right|,$$ giving us

$$ \left| \sqrt{d} - \dfrac{p}{q} \right| \geq \dfrac{1}{q^2} \left(\dfrac{1}{2\sqrt{d} + \left| \sqrt{d} - \frac{p}{q} \right|} \right).$$

Writing $ t := \left| \sqrt{d} - \dfrac{p}{q} \right| $, this is $ t \geq \dfrac{1}{q^2 (2\sqrt{d} + t)} $. So $ q^2 t^2 + q^2 2\sqrt{d} t - 1 \geq 0 $.

Quadratic on LHS has roots $ \dfrac{-q \sqrt{d} \pm \sqrt{ q^2 d + 1 }}{q} $, so the inequality becomes $$ \left(t-\frac{-q\sqrt{d} - \sqrt{q^2 d+1}}{q} \right) \left( t - \frac{-q\sqrt{d}+\sqrt{q^2 d+1}}{q} \right) \geq 0.$$

The first factor is anyways $ > 0 $, giving $ t \geq \dfrac{-q\sqrt{d} + \sqrt{q^2 d + 1}}{q} $. Now RHS $$ \dfrac{-q\sqrt{d} + \sqrt{q^2 d + 1}}{q} = \dfrac{1}{q(q\sqrt{ d} + \sqrt{q^2 d + 1})}=\dfrac{1}{q^2\left(\sqrt{d} + \sqrt{d + \dfrac{1}{q^2}}\right)}\geq \dfrac{1}{q^2(\sqrt{d} + \sqrt{d+1})}= \dfrac{\sqrt{d+1}-\sqrt{d}}{q^2}$$


So finally, we have

$$ \left| \sqrt{d} - \dfrac{p}{q} \right| \geq \dfrac{\sqrt{d+1}-\sqrt{d}}{q^2} $$

for any integers $ p, q $ with $ q > 0 $.